Difference between revisions of "1997 PMWC Problems/Problem I1"

(Solution)
m
Line 5: Line 5:
 
<math>\frac{29 \cdot 28 + 27}{28} \cdot \frac{27 \cdot 15 + 14}{15} = \frac{839 \cdot 419}{420} = \frac{839(420 - 1)}{420} = 839 - \frac{839}{420} = 837 \frac{1}{420}</math>
 
<math>\frac{29 \cdot 28 + 27}{28} \cdot \frac{27 \cdot 15 + 14}{15} = \frac{839 \cdot 419}{420} = \frac{839(420 - 1)}{420} = 839 - \frac{839}{420} = 837 \frac{1}{420}</math>
  
== See also ==
+
== See Also ==
 
{{PMWC box|year=1997|before=First question|num-a=I2}}
 
{{PMWC box|year=1997|before=First question|num-a=I2}}
  
 
[[Category:Introductory Algebra Problems]]
 
[[Category:Introductory Algebra Problems]]

Revision as of 16:03, 15 May 2012

Problem

Evaluate $29 \frac{27}{28} \times 27\frac{14}{15}$.

Solution

$\frac{29 \cdot 28 + 27}{28} \cdot \frac{27 \cdot 15 + 14}{15} = \frac{839 \cdot 419}{420} = \frac{839(420 - 1)}{420} = 839 - \frac{839}{420} = 837 \frac{1}{420}$

See Also

1997 PMWC (Problems)
Preceded by
First question
Followed by
Problem I2
I: 1 2 3 4 5 6 7 8 9 10 11 12 13 14 15
T: 1 2 3 4 5 6 7 8 9 10